Difference between revisions of "1966 AHSME Problems/Problem 30"

(Solution)
(Solution)
 
Line 5: Line 5:
  
 
== Solution ==
 
== Solution ==
<math>\fbox{D}</math>
+
Since this is a quartic equation, there are going to be <math>4</math> solutions.
 +
By Vieta's formulas, since the <math>x^3</math> term is <math>0</math>, the sum of the roots is also <math>0</math>. Therefore, the 4th root of this polynomial is <math>-6</math>.
 +
Lastly, by Vieta's Formulas, <math>a+c=-(1\cdot2+1\cdot3+1\cdot-6+2\cdot3+2\cdot-6+3\cdot-6)+1\cdot2\cdot3\cdot-6=-25-36=-61=
 +
\fbox{D}</math>
  
 
== See also ==
 
== See also ==

Latest revision as of 15:38, 26 July 2021

Problem

If three of the roots of $x^4+ax^2+bx+c=0$ are $1$, $2$, and $3$ then the value of $a+c$ is:

$\text{(A) } 35 \quad \text{(B) } 24 \quad \text{(C) } -12 \quad \text{(D) } -61 \quad \text{(E) } -63$

Solution

Since this is a quartic equation, there are going to be $4$ solutions. By Vieta's formulas, since the $x^3$ term is $0$, the sum of the roots is also $0$. Therefore, the 4th root of this polynomial is $-6$. Lastly, by Vieta's Formulas, $a+c=-(1\cdot2+1\cdot3+1\cdot-6+2\cdot3+2\cdot-6+3\cdot-6)+1\cdot2\cdot3\cdot-6=-25-36=-61= \fbox{D}$

See also

1966 AHSME (ProblemsAnswer KeyResources)
Preceded by
Problem 29
Followed by
Problem 31
1 2 3 4 5 6 7 8 9 10 11 12 13 14 15 16 17 18 19 20 21 22 23 24 25 26 27 28 29 30
All AHSME Problems and Solutions

The problems on this page are copyrighted by the Mathematical Association of America's American Mathematics Competitions. AMC logo.png